Integral of hyperbolic function

Click For Summary
The integral of the hyperbolic function presented involves evaluating \(\int^4_3 \frac{1}{\sqrt{3x^2-6x+1}}\,dx\) by completing the square. The expression simplifies to \(\frac{1}{\sqrt 3} \left[\cosh^{-1}\frac{3\sqrt 3}{\sqrt 2}-\cosh^{-1}\sqrt 6\right]\). However, there is a discrepancy between the calculated result and the answer from notes, leading to confusion about the correctness of the notes. The final evaluations yield different numerical results, prompting speculation that either the question or the reference answer may be incorrect. The discussion highlights the challenges in verifying integral solutions involving hyperbolic functions.
DryRun
Gold Member
Messages
837
Reaction score
4
Homework Statement
\int^4_3 \frac{1}{\sqrt{3x^2-6x+1}}\,.dx

The attempt at a solution
I complete the square for the quadratic:
\sqrt{3x^2-6x+1}<br /> \\=\sqrt{3(x^2-2x+\frac{1}{3})}<br /> \\=\sqrt 3 \times \sqrt{(x-1)^2-\frac{2}{3}}
\int^4_3 \frac{1}{\sqrt{3x^2-6x+1}}\,.dx<br /> \\=\frac{1}{\sqrt 3}\int^4_3 \frac{1}{\sqrt{(x-1)^2-\frac{2}{3}}}\,.dx<br /> \\=\frac{1}{\sqrt 3} \left[\cosh^{-1}\frac{\sqrt 3(x-1)}{\sqrt 2}\right]^{x=4}_{x=3}<br /> \\=\frac{1}{\sqrt 3} \left[\cosh^{-1}\frac{3\sqrt 3}{\sqrt 2}-\cosh^{-1}\sqrt 6\right]
I already simplified it but it doesn't agree with the final answer:
\frac{1}{\sqrt 3}\ln \left(\frac{15+\sqrt {219}}{12+\sqrt {138}}\right)
 
Physics news on Phys.org
hi sharks! :smile:

cosh-1x = ln(ecosh-1x)

= ln(cosh(cosh-1x) + sinh(cosh-1x))

= ln (x + √(x2 - 1)) :wink:
 
Hi tiny-tim :smile:

The above expression is what i used to expand:
\frac{1}{\sqrt 3} \left [\cosh^{-1}\frac{3\sqrt 3}{\sqrt 2}-\cosh^{-1}\sqrt 6\right]<br /> \\=\frac{1}{\sqrt 3} \ln \left[\frac {3\sqrt 3+5}{\sqrt{12}+\sqrt{10}}\right]=0.4625025064
But if i evaluate the answer from my notes, i get:
\frac{1}{\sqrt 3}\ln \left (\frac{15+\sqrt {219}}{12+\sqrt {138}}\right)=0.1310541888
Since the answers are not the same, I'm thinking that maybe the answer in my notes is wrong?
 
Last edited:
well that's obviously …
\frac{1}{\sqrt 3} \left[\cosh^{-1}\frac{\sqrt 3(x+1)}{\sqrt 2}\right]^{x=4}_{x=3}

… either the question or the answer is wrong
 
Question: A clock's minute hand has length 4 and its hour hand has length 3. What is the distance between the tips at the moment when it is increasing most rapidly?(Putnam Exam Question) Answer: Making assumption that both the hands moves at constant angular velocities, the answer is ## \sqrt{7} .## But don't you think this assumption is somewhat doubtful and wrong?

Similar threads

  • · Replies 105 ·
4
Replies
105
Views
6K
Replies
4
Views
2K
  • · Replies 5 ·
Replies
5
Views
2K
Replies
4
Views
2K
  • · Replies 10 ·
Replies
10
Views
2K
  • · Replies 22 ·
Replies
22
Views
3K
  • · Replies 10 ·
Replies
10
Views
2K
  • · Replies 14 ·
Replies
14
Views
2K
  • · Replies 5 ·
Replies
5
Views
3K
  • · Replies 4 ·
Replies
4
Views
2K